Question

In: Statistics and Probability

What is the upper limit of the PDF y-axis? CDF y-axis? I think it is infinity...

What is the upper limit of the PDF y-axis? CDF y-axis? I think it is infinity and 1 but I want to double check. Please also explain conceptually why this is the case. I know the area under the curve of PDF = CDF.

Solutions

Expert Solution

Concepts of CDF and PDF


Related Solutions

When do I use: Geometric cdf Geometric pdf binomial cdf Binomial pdf Thank you
When do I use: Geometric cdf Geometric pdf binomial cdf Binomial pdf Thank you
What is the difference between a pdf and a cdf? How are they related? Explain this...
What is the difference between a pdf and a cdf? How are they related? Explain this as you would to someone who wasn't in class when I did. Assume they have taken Calculus 2. You can use text, pictures, a video, a dance, a song... whatever you want. Be creative! Submit your explanation as one file (a pdf for text or image, a single video or audio file etc.)
1) finding the volume of solid whose upper limit is the surface f (x, y) =...
1) finding the volume of solid whose upper limit is the surface f (x, y) = 4xe^y and which lower limit is the region r. where r is the triangle limited by y = 2x; y = 2; x = 0.
Given the data below, what is the upper control limit for the Moving Range control chart?...
Given the data below, what is the upper control limit for the Moving Range control chart? Please enter your answer with at least 4 significant digits. Data: Observation 132.4654 118.9743 124.6528 130.3063 144.1334 118.4584 135.1793 115.2477 138.6022 104.4394 128.8716 133.7959 113.2013 120.4394 142.4859 123.592 135.9269 123.5473 139.3181 138.444 135.2332 125.4248 123.4138 138.3623 120.3414 126.7968 126.2955 138.4302 133.506 115.1217 119.105 136.3829 142.9304 146.3562 115.0906 147.2637 143.7961 140.0805
let the continuous random variables X and Y have the joint pdf: f(x,y)=6x , 0<x<y<1 i)...
let the continuous random variables X and Y have the joint pdf: f(x,y)=6x , 0<x<y<1 i) find the marginal pdf of X and Y respectively, ii) the conditional pdf of Y given x, that is fY|X(y|x), iii) E(Y|x) and Corr(X,Y).
Holding everything constant, what would be the upper limit of the 90% confidence interval for the population mean?
In a sample of n=3,600 students, the average GPA was x⎯⎯=3.5 with a sample variance of s2=0.25. Calculate the 98% confidence interval for the population GPA μ Holding everything constant, what would be the upper limit of the 90% confidence interval for the population mean? (round to two digits)   3.48   3.49   3.51   3.52
Why can't I use the sum of force(about y axis) =0 at point P? I used...
Why can't I use the sum of force(about y axis) =0 at point P? I used this method to get the tension in the cord at first of solution, but gotten wrong answer. http://www.chegg.com/homework-help/mechanics-of-materials-8th-edition-chapter-1-problem-6p-solution-9780136022305?trackid=1b3aa2d6&strackid=4d3f9551&ii=1
What do you think...should there be a limit or cap on the maximum amount that a...
What do you think...should there be a limit or cap on the maximum amount that a jury awards a victim of medical malpractice? If Yes...why? If No...why not?
If good X is measured on the horizontal axis and good Y on the vertical, what...
If good X is measured on the horizontal axis and good Y on the vertical, what can you say about the preferences of someone whose indifference curves are parallel to the Y axis? positively sloped with more desirable indifference curves as one moves to the right? negatively sloped with more desirable indifference curves as one moves to the left? L-shaped with one line of the L shape parallel to X axis and the other line of the L shape parallel...
2. Let X be exponential with rate lambda. What is the pdf of Y = X^0.5?...
2. Let X be exponential with rate lambda. What is the pdf of Y = X^0.5? How about Y = X^3? Contrast the complexity of this result to transformation of a discrete random variable.
ADVERTISEMENT
ADVERTISEMENT
ADVERTISEMENT